Autor Tema: Paradoja sobre la distribución de los números primos

0 Usuarios y 1 Visitante están viendo este tema.

04 Mayo, 2023, 09:21 pm
Leído 313 veces

Joseferm

  • $$\Large \color{#6a84c0}\pi$$
  • Mensajes: 32
  • Karma: +0/-0
  • Sexo: Masculino
Buenos días.

Sabemos que los números primos se van haciendo cada vez más escasos a medida que vamos recorriendo los enteros, de forma que incluso podemos encontrar a voluntad "lagunas" inmensamente grandes  sin absolutamente ningún primo .

 Conocemos el teorema de los números primos de  Gauss  según el cual  si \( \pi(x) \) es el número de primos menores que x , entonces  \( \pi(x)\rightarrow{} x/ Ln x \)  cuando \( x\rightarrow{}\infty \).  También conocemos las mejoras introducidas en ese conteo de primos por  Dirichlet, Chebychev y Riemann.

Según todo ello, de forma intuitiva cuando uno toma un número N cualquiera ya sabe que el número de primos inferiores a N es mayor que el número de primos comprendidos entre N y 2N.  Y si el número N es muy, muy alto, uno espera que los primos inferiores a N sean muchísimos más que los comprendidos entre N y 2N.


Y aquí viene lo que para mí desde luego es una paradoja.  Si voy dando valores cada vez mayores a N  y calculo el cociente entre los comprendidos entre N y 2N  y los primos inferiores a N, vemos que ese cociente  va creciendo desde valores próximos a 0.80  hasta valores de 0.99......  de forma que según los valores expuestos en la tabla y en la gráfica que adjunto ese valor del cociente tiende a 1 cuando N   tiende a  \( \infty  \)


Esto significa que a medida que N crece, la cantidad de primos existentes entre 1 y N  y la que hay entre N y 2N tienden a igualarse y que si tomamos  un N inmensamente grande ( estoy pensando en   \( \infty / 2 \)  entonces el cociente llega a ser 1  y por lo tanto  hay tantos primos por debajo de \( \infty / 2 \)  como por encima de  \( \infty / 2 \) , lo cual contradice frontalmente todo lo que sé sobre la disminución de la densidad de primos a medida que avanzamos en la recta numérica. 

A mí, personalmente me resulta totalmente contrario a la intuición.

04 Mayo, 2023, 10:00 pm
Respuesta #1

Luis Fuentes

  • el_manco
  • Administrador
  • Mensajes: 56,049
  • País: es
  • Karma: +0/-0
Hola

 Yo no veo ninguna paradoja. Teniendo en cuenta que aproximadamente \( \pi(x)\approx f(x)=\dfrac{x}{ln(x)} \), lo que estamos diciendo es que:

\(  \displaystyle\lim_{x \to{+}\infty}{}\dfrac{f(2x)-f(x)}{f(x)}=1 \)

 Pero:

\( \dfrac{f(2x)-f(x)}{f(x)}=\dfrac{f(2x)}{f(x)}-1=\dfrac{2ln(x)}{ln(2x)}-1=\dfrac{2ln(x)}{ln(x)-1}-1=1+\dfrac{2}{ln(x)-1}\to 1 \)

 Y la cosa es que aunque una función disminuya respecto a otra su cociente puede igualmente tender a una constante (a uno por ejemplo). Por ejemplo las funciones \( f(x)=x \) y \( g(x)=x-ln(x) \), cumplen que la primera es cada vez más grande que la segunda (y de hecho \( f(x)-g(x)\to \infty \)) pero el límite del cociente es uno.

Saludos.

04 Mayo, 2023, 11:42 pm
Respuesta #2

Joseferm

  • $$\Large \color{#6a84c0}\pi$$
  • Mensajes: 32
  • Karma: +0/-0
  • Sexo: Masculino
Buenas noches Luis.

Completamente de acuerdo en lo que expones, porque matemáticamente  en el límite así es.

Cuando digo paradoja, me refiero a que dado que los números primos se van volviendo más y más  escasos cada vez, es contrario a la intuición que en el límite , cuando N se vuelve tan sumamente grande que llega por decirlo así a la mitad de infinito, haya tantos primos a su izquierda como a su derecha.  O sea, yo al menos, tenía la falsa impresión, o la "falsa ilusión"  de que para ese N tan grande, habría siempre muchísimos más primos menores que N,  que primos mayores que N .

Un saludo.

05 Mayo, 2023, 08:43 am
Respuesta #3

Luis Fuentes

  • el_manco
  • Administrador
  • Mensajes: 56,049
  • País: es
  • Karma: +0/-0
Hola

Cuando digo paradoja, me refiero a que dado que los números primos se van volviendo más y más  escasos cada vez, es contrario a la intuición que en el límite , cuando N se vuelve tan sumamente grande que llega por decirlo así a la mitad de infinito, haya tantos primos a su izquierda como a su derecha. O sea, yo al menos, tenía la falsa impresión, o la "falsa ilusión"  de que para ese N tan grande, habría siempre muchísimos más primos menores que N,  que primos mayores que N .

Pero es que la expresión en rojo tiene una interpretación muy abierta. La diferencia de primos a derecha e izquierda es cada vez más grande y de hecho tiende a infinito; sin embargo el ratio de unos y otros tiende a uno. Son cosas compatibles.

Te puse otro ejemplo de ese fenómeno aquí:

Y la cosa es que aunque una función disminuya respecto a otra su cociente puede igualmente tender a una constante (a uno por ejemplo). Por ejemplo las funciones \( f(x)=x \) y \( g(x)=x-ln(x) \), cumplen que la primera es cada vez más grande que la segunda (y de hecho \( f(x)-g(x)\to \infty \)) pero el límite del cociente es uno.

y se pueden poner otros muchos.

Saludos.

05 Mayo, 2023, 09:53 am
Respuesta #4

feriva

  • $$\Large \color{#a53f54}\pi\,\pi\,\pi\,\pi\,\pi\,\pi\,\pi$$
  • Mensajes: 11,330
  • País: es
  • Karma: +1/-0
  • Sexo: Masculino
Buenos días.

Sabemos que los números primos se van haciendo cada vez más escasos a medida que vamos recorriendo los enteros, de forma que incluso podemos encontrar a voluntad "lagunas" inmensamente grandes  sin absolutamente ningún primo .

 Conocemos el teorema de los números primos de  Gauss  según el cual  si \( \pi(x) \) es el número de primos menores que x , entonces  \( \pi(x)\rightarrow{} x/ Ln x \)  cuando \( x\rightarrow{}\infty \).  También conocemos las mejoras introducidas en ese conteo de primos por  Dirichlet, Chebychev y Riemann.

Según todo ello, de forma intuitiva cuando uno toma un número N cualquiera ya sabe que el número de primos inferiores a N es mayor que el número de primos comprendidos entre N y 2N.  Y si el número N es muy, muy alto, uno espera que los primos inferiores a N sean muchísimos más que los comprendidos entre N y 2N.


Y aquí viene lo que para mí desde luego es una paradoja.  Si voy dando valores cada vez mayores a N  y calculo el cociente entre los comprendidos entre N y 2N  y los primos inferiores a N, vemos que ese cociente  va creciendo desde valores próximos a 0.80  hasta valores de 0.99......  de forma que según los valores expuestos en la tabla y en la gráfica que adjunto ese valor del cociente tiende a 1 cuando N   tiende a  \( \infty  \)


Esto significa que a medida que N crece, la cantidad de primos existentes entre 1 y N  y la que hay entre N y 2N tienden a igualarse y que si tomamos  un N inmensamente grande ( estoy pensando en   \( \infty / 2 \)  entonces el cociente llega a ser 1  y por lo tanto  hay tantos primos por debajo de \( \infty / 2 \)  como por encima de  \( \infty / 2 \) , lo cual contradice frontalmente todo lo que sé sobre la disminución de la densidad de primos a medida que avanzamos en la recta numérica. 

A mí, personalmente me resulta totalmente contrario a la intuición.

La “paradoja” vista intuitivamente con unos ejemplos viene a ser ésta:

Razón: \( \dfrac{5}{4}=1,25 \). Diferencia: \( 5-4=1 \).

...

Razón: \( \dfrac{5000}{4500}=1,111... \). Diferencia: \( 5000-4500=500 \)

...

Vemos que la razón 5000/5400 se acerca más a 1 que la razón 5/4; decimos que 5000 y 4500 son más “parecidos” que 4 y 5. Sin embargo, eso no quiere decir que la diferencia entre 5400 y 5000 sea menor que la que existe entre 4 y 5; ni mucho menos. Lo que ocurre es que la unidad se queda “pequeña” por comparación cuando los números son muy grandes.

Esto significa que, en realidad, por mucho que te vayas al infinito, la cantidad nunca es exacta; al contrario, la diferencia es cada vez más grande. La diferencia, comparando la cantidad de primos hasta un x grande y hasta un x pequeño, lejos de tender a 1, tiende a infinito, lo cual es trivial y muy intuitivo ya desde el principio: aquí 1,2,3, hay dos primos, aquí 1,2,3,4,5,6,7,8,9,10,11,12,13, hay seis... y si ahora piensas en que existen infinitos primos... pues no cabe duda.

Con eso no me estoy refiriendo a los intervalos n y 2n, pero tiene que ver con lo que te despista:

\( \dfrac{10}{log(10)}=4 \)

y así es, la cantidad real de primos coincide con esa, son 2,3,5,7. Habiendo tomado un n pequeño obtenemos la cantidad exacta.

\( \dfrac{10000}{log(10000)}=1085,736... \)

Pero la cantidad real hasta 10000 es 1229, o sea, en torno 144 primos más de lo que dice la parte entera. Si el número x tiende a infinito, en vez de tender a 144 primos menos respecto de la cantidad real, tiende a infinitos primos menos de los que en realidad hay.


Una cosa es la proporción y otra distinta la diferencia.

Saludos.

05 Mayo, 2023, 09:14 pm
Respuesta #5

Joseferm

  • $$\Large \color{#6a84c0}\pi$$
  • Mensajes: 32
  • Karma: +0/-0
  • Sexo: Masculino
Muy buena respuesta, Feriva.
Gracias